Verbal questions from any Manhattan Prep GMAT Computer Adaptive Test. Topic subject should be the first few words of your question.
Shriram
 
 

CAT #2 CR: The pace of technological development brings

by Shriram Sun Jun 22, 2008 10:49 pm

Analyst: The pace of technological development brings a constant stream of new devices to the market, and many of them enjoy commercial success. But announcing new technology too soon after the introduction of a successful device can backfire. Once consumers hear about the new device, they may stop buying the one currently on sale. So, if a company wishes to announce the upcoming sale of a new device, it should wait until purchases of the old device have begun to decline.

Which of the following, if true, would best support the analyst's main assertion?
A. New technology often becomes less expensive after an initial surge in sales.
B. Media outlets, such as television programs and magazines, often report on the planned introduction of new devices while the sales of old devices are still strong.
C. Many consumers are unable to determine whether new technology is superior to current
technology.
D. Surveys have shown that some consumers make only one or two technology purchases per year, whereas others make more frequent purchases.
E. Consumers tend to be loyal to technology companies whose products they enjoy using.

The main conclusion is the last sentence of the question stem. The correct answer choice is shown as B. However, B does not state that media outlets report on the planned introduction of new devices when the sales of old devices are still strong. Doesn't this weaken (or at least have no effect on) the conclusion rather than strengthen it? I know I am probably missing something basic here, and so would appreciate any help with this.

thanks,
-Shriram.
Shriram
 
 

typo

by Shriram Mon Jun 23, 2008 9:12 am

Sorry, I realised the mistake in my earlier post. What I meant is that "B states that the media outlets and magazines do NOT wait until sales of earlier products have begun to decline, before introducing new products, but rather they start introducing them when sales of existing products are still strong".

-Shriram.
texas2904
 
 

by texas2904 Fri Jul 11, 2008 1:43 am

I was about to post the same question. In my opinion, if the media has already released information on new products then the conclusion that companies should wait until sales decline is weakened. People generally read rumors about new products on line long before the company releases info. As an example, the iphone stopped selling because people found out about the new one being better long before apple made that info public. By the time they announced it, sales had already slowed down because of the info leak, so why couldn't they have announced it months earlier? I stared at this problem for a while and eventually chose C thinking that if new tech were actually inferior, people wouldn't be able to tell, and the premises about people waiting to buy the new tech over old tech would still be true. That would not maybe strengthen the conclusion a little bit, but not much.
RonPurewal
Students
 
Posts: 19744
Joined: Tue Aug 14, 2007 8:23 am
 

by RonPurewal Fri Jul 18, 2008 4:03 am

i see the point in the last couple of posts, but these sorts of "problems" / "incongruities" could easily show up on official questions as well.

here's the clincher when you have to decide something like this: pick the choice that stays as CLOSE to the logic / conclusion of the passage as possible.

the problem with choice c is that it introduces a new consideration - "superiority" - that is COMPLETELY unaddressed in, and therefore irrelevant to, the passage. you have to make all kinds of assumptions to push this answer choice into even the darkest corners of the argument's scope: in particular, (1) you have to introduce the assumption that consumers make tech purchasing decisions based on "superiority", AND (2) even if you allow this assumption (note that allowing random assumptions is never a good idea), you still have no idea how the "superiority" angle is going to affect purchasing decisions! you'd have to introduce even more assumptions along the lines of "the new version is superior / inferior to the old version".
bad bad bad.

you are right that choice b requires the assumption that media coverage of new devices will follow the official announcement of those devices. this may be somewhat of an assumption, but it's absolutely trifling relative to the enormous assumptions detailed above for choice c.
what's more, this assumption is clearly relevant to the argument. don't forget that the vast majority of wrong answers on weaken/strengthen problems are wrong because they're totally irrelevant to the argument. if you can find a weaken/strengthen answer choice that is CLEARLY related to the information in the passage, then you're halfway there!

one more consideration: your reasoning against choice b is weaker than it may at first seem. while it may be true that the media publish speculative reports about "leaked" products, (1) you have to assume that that happens, always a dangerous thing, and (2) for average products, the amount and effect of such speculative coverage is almost certainly very, very small.
the fact that you used the iphone - a very exceptional device, one that defies just about every market norm out there in some way - as your example is telling: such reasoning is unlikely to extend to run-of-the-mill technological products such as the ones presumably treated by the passage.
texas2904
 
 

by texas2904 Mon Jul 21, 2008 8:46 pm

Ron,

Thanks for replying. I understand what you are saying about the assumption, but I don't see how the answer supports the conclusion. My main problem is failing to see how the media acting against what the conclusion is suggesting can be considered support for it. The answer choice says nothing about sales.

Thanks.
esledge
Forum Guests
 
Posts: 1181
Joined: Tue Mar 01, 2005 6:33 am
Location: St. Louis, MO
 

by esledge Sun Sep 07, 2008 6:09 pm

Here’s how I think of it:

(A) Cost--unrelated
(B) "Planned introduction" isn’t necessarily equal to "upcoming sale" in terms of immediacy. A media outlet might report on the planned introduction of a hybrid vehicle in 2011, and that would drive interest in advance of late 2010 when the car company could announce the "upcoming sale." Also, "sales are still strong" doesn’t necessarily mean that sales have not "begun to decline." In fact, you might argue that the use of the word "still" suggests that the decline has begun.
(C) Technical superiority--unrelated
(D) Frequency of purchase varies among consumers--unrelated
(E) Consumer loyalty--unrelated

In summation, I can definitely see your points Shiriam and texas2904. I’m not wild about (B) as a strengthener, either. But it is the "best support" among the choices given. The big take-away is Ron's point: (B) is the choice closest to the argument situation, without the addition of any new factors.
Emily Sledge
Instructor
ManhattanGMAT